Linear transformations formula

Click For Summary
The discussion revolves around finding the formula for a linear transformation T from R^3 to R^2, given specific basis vectors and their transformations. The user struggles with expressing a vector in terms of the basis and solving for the coefficients c1, c2, and c3. They mention that the expected answers for these coefficients seem unclear and attempt to use matrix row reduction without success. The conversation highlights the importance of determining the transformation matrix to facilitate calculations for any input vector. Ultimately, the focus is on understanding how to derive the transformation matrix and apply it to find the output for specific inputs.
Aerosion
Messages
52
Reaction score
0
Linear transformations...

Homework Statement



Can't figure these things out for my life. Seriously. Here's an example.

consider the basis S={v1, v2, v3} for R^3 where v1=(1,2,1), v2=(2,9,0) and v3=(3,3,4) and let T:R^3-->R^2 be the linear transformation such that:

T(v1)=(1,0) T(v2)=(-1,1) T(v3)=(0,1)

Find formula for T(x1,x2,x3) and use it to find T(7,13,7)

Homework Equations





The Attempt at a Solution




So here I am starting it, writing c1(1,2,1)+c2(2,9,0)-C3(3,3,0) to make C1+2C2+3C3, 2C1+9C2+3C3, C1+C3

c1+2c2+3c3=x1
2c1+4c2+3c3=x2
c1+ +4c3=x3

Now, I'm having trouble solving for c1, c2, and c3. The answers seem to be c1=36x1+8x2+21x3, c2=5x1-x2-3x3, c3=9x1-2x2-5x3. I don't know where those numbers came from. (I was guessing putting it into a matrix and use row reduction, but...no). Hmm? Help?
 
Physics news on Phys.org
So the question is asking for the matrix that sends (1,2,1), ..., to their respective vectors in lR^2.

A v1 = ?
A v2 = ?
A v3 = ?

It seems like you are trying to find linear combinations of the vectors that will give the transformations. I don't think that is going to work.

If you can find the matrix that performs the transformations, you can use matrix multiplication to find the vectors corresponding to certain coordinates (c1,c2).
 
Last edited:
Question: A clock's minute hand has length 4 and its hour hand has length 3. What is the distance between the tips at the moment when it is increasing most rapidly?(Putnam Exam Question) Answer: Making assumption that both the hands moves at constant angular velocities, the answer is ## \sqrt{7} .## But don't you think this assumption is somewhat doubtful and wrong?

Similar threads

  • · Replies 7 ·
Replies
7
Views
3K
  • · Replies 5 ·
Replies
5
Views
5K
  • · Replies 10 ·
Replies
10
Views
2K
  • · Replies 26 ·
Replies
26
Views
3K
  • · Replies 1 ·
Replies
1
Views
2K
  • · Replies 1 ·
Replies
1
Views
1K
Replies
2
Views
2K
Replies
1
Views
2K
  • · Replies 2 ·
Replies
2
Views
2K
  • · Replies 2 ·
Replies
2
Views
2K